evaluate log25+log8 _log2​

Answers

Answer 1

Step-by-step explanation:

log 25 + log 8 = log (25 × 8) = log 200

log 200 - log 2 = log (200 ÷ 2) = log 100

logarithm base 10 of 100 is 2.


Related Questions

Write the equation of the line (in y = ax+b form) in the
graph below. do not use any spaces when typing your
answer

Answers

Answer:

y=3x-8

Step-by-step explanation:


Out of the 30 days in June, you run 13 days and swim 6 days. You lift weights 7 days
and ride a bike 4 days. If you make a pie chart to show this data, how many degrees
of the circle should represent swimming?

Answers

9514 1404 393

Answer:

  72°

Step-by-step explanation:

In a properly constructed pie chart, the proportion of the circle devoted to each section is the same as the proportion of that section to the the whole.

  (6 swim days)/(30 days) = (swim pie angle)/(360°)

Multiply by 360° to find ...

  swim pie angle = (360°)(6/30) = 72°

72° of the circle should represent swimming.

What is the measure of Arc E B C?

Answers

Answer:

The measure of arc (EBC) is 220°.



A square has a perimeter of 36 cm.
What is the length of each side?

Answers

9 cm i hope this helps

Answer:

Step-by-step explanation:

The perimeter of a square has a formula P = s + s + s + s or just P = 4s where s is the length of a side. If this perimeter has a number value, we can plug it in and solve for the length of each side, like this:

36 = 4s so

s = 9 cm. And there you go!

In which section of the number line is 32−−√?

Answers

where's the number line?

maybe u can attach it at the comments:)

Answer:

Section B

Step-by-step explanation:

x/3 + 8 = 23 . Find the value of x

Answers

Answer:

45

Step-by-step explanation:

x/3 + 8 =23

=> x/3 = 23-8

=> x/3 = 15

=> x = 15 x 3

x = 45

Answer:

x=45

Step-by-step explanation:

To find x isolate the variables by using the properties of equality. First, subtract 8 from both sides, [tex]\frac{x}{3} = 15[/tex]. Then multiply both sides by 3, [tex]x=45[/tex]. Finally, to check you can plug 45 back into the equation, [tex]\frac{45}{3} +8=23[/tex]. Next, solve this equation and get 23=23, since this is a true statement 45 is correct.

SOMEONE PLEASE HELPPPPPPPPPP

Answers

Answer:

the formula for the volume of a square pyramid is V=a^2*h/3

so if we just put in the data

12^2*10/3

144*10/3

480

so the answer is B

Hope This Helps!!!

What is
4
7
as a decimal rounded to 3 decimal places?

Answers

Answer:

4/7 as a decimal is 0.57142857142857.

(17)^ 3 *(289)^ -6 =(17)^ 2m - 1​

Answers

Answer:

m = -4

Step-by-step explanation:

The given expression is :

[tex](17)^{3}\cdot(289)^{-6}=(17)^{2m-1}[/tex]

We need to find the value of m

We know that, 17² = 289

So,

[tex](17)^{3}\cdot(17^2)^{-6}=(17)^{2m-1}\\\\(17)^{3}\cdot(17)^{-12}=(17)^{2m-1}[/tex]

Also,[tex]x^ax^b=x^{a+b}[/tex]

So,

[tex]17^{3-12}=17^{2m-1}\\\\17^{-9}=17^{2m-1}\\\\\implies -9=2m-1\\\\-9+1=2m\\\\-8=2m\\\\m=-4[/tex]

So, the value of m is equal to -4.

How to find the frequency in a histogram

Answers

Answer:Para preparar la tabla de frecuencia para un histograma, es necesario primero establecer el número de intervalos que se desea tener.

Luego, se debe determinar el ancho común de los intervalos (interval width). Para esto, se calcula la diferencia del dato mayor y el dato menor, y se divide entre el número de intervalos deseados. Este resultado se redondea al entero mayor más cercano.

Por ejemplo, si se desea tener 10 intervalos y encontramos que el dato mayor es 35 y el menor es 12,

Step-by-step explanation:

is (12, 4), (36, 12), (-6, -2), (21, 7) directly proportional?​

Answers

Answer:

I think yes

Step-by-step explanation:

12/4=3

36/12=3

-6/-2=3

and so on

what is the answer to the problem

Answers

Answer:

Step-by-step explanation:

29 is 6 more than k

Write it as an mathematical equation!

29 = 6 + k

Then solve for k

K = 29 - 6
K = 23

Find the product: 3/4 x 2/3. What's the product? 5/7, 6/12, 5/12, 6/7. What is it please help me!!!!

Answers

Answer:

1/2

Step-by-step explanation:

3/4 x 2/3

= 1/2

Consider the line 4x + 8y = -4.
a) What is the slope of a line perpendicular to this line?

b) What is the slope of a line parallel to this line?

Answers

a) -2

a slope of a line perpendicular is it’s opposite and reciprocal.

Therefore you want to find the slope. You can do this by writing it in intercept form, or just knowing A/B since it’s in standard form.

slope form: y= mx + b

8y = -4 -4x

y = -1/2x -1/2

So the slope is -1/2 so it’s opposite and reciprocal would be 2. So a slope of a line perpendicular is -2.

b) -1/2

A line parallel would have the same slope as your equation. Since you know the slope -1/2 you know whatever your b value is, that line would be parallel as long as it has the same slope.

(a) Determine a cubic polynomial with integer coefficients which has $\sqrt[3]{2} + \sqrt[3]{4}$ as a root.

(b) Prove that $\sqrt[3]{2} + \sqrt[3]{4}$ is irrational.

Answers

Answer:

(a) [tex]x\³ - 6x - 6[/tex]

(b) Proved

Step-by-step explanation:

Given

[tex]r = $\sqrt[3]{2} + \sqrt[3]{4}$[/tex] --- the root

Solving (a): The polynomial

A cubic function is represented as:

[tex]f = (a + b)^3[/tex]

Expand

[tex]f = a^3 + 3a^2b + 3ab^2 + b^3[/tex]

Rewrite as:

[tex]f = a^3 + 3ab(a + b) + b^3[/tex]

The root is represented as:

[tex]r=a+b[/tex]

By comparison:

[tex]a = $\sqrt[3]{2}[/tex]

[tex]b = \sqrt[3]{4}$[/tex]

So, we have:

[tex]f = ($\sqrt[3]{2})^3 + 3*$\sqrt[3]{2}*\sqrt[3]{4}$*($\sqrt[3]{2} + \sqrt[3]{4}$) + (\sqrt[3]{4}$)^3[/tex]

Expand

[tex]f = 2 + 3*$\sqrt[3]{2*4}*($\sqrt[3]{2} + \sqrt[3]{4}$) + 4[/tex]

[tex]f = 2 + 3*$\sqrt[3]{8}*($\sqrt[3]{2} + \sqrt[3]{4}$) + 4[/tex]

[tex]f = 2 + 3*2*($\sqrt[3]{2} + \sqrt[3]{4}$) + 4[/tex]

[tex]f = 2 + 6($\sqrt[3]{2} + \sqrt[3]{4}$) + 4[/tex]

Evaluate like terms

[tex]f = 6 + 6($\sqrt[3]{2} + \sqrt[3]{4}$)[/tex]

Recall that: [tex]r = $\sqrt[3]{2} + \sqrt[3]{4}$[/tex]

So, we have:

[tex]f = 6 + 6r[/tex]

Equate to 0

[tex]f - 6 - 6r = 0[/tex]

Rewrite as:

[tex]f - 6r - 6 = 0[/tex]

Express as a cubic function

[tex]x^3 - 6x - 6 = 0[/tex]

Hence, the cubic polynomial is:

[tex]f(x) = x^3 - 6x - 6[/tex]

Solving (b): Prove that r is irrational

The constant term of [tex]x^3 - 6x - 6 = 0[/tex] is -6

The divisors of -6 are: -6,-3,-2,-1,1,2,3,6

Calculate f(x) for each of the above values to calculate the remainder when f(x) is divided by any of the above values

[tex]f(-6) = (-6)^3 - 6*-6 - 6 = -186[/tex]

[tex]f(-3) = (-3)^3 - 6*-3 - 6 = -15[/tex]

[tex]f(-2) = (-2)^3 - 6*-2 - 6 = -2[/tex]

[tex]f(-1) = (-1)^3 - 6*-1 - 6 = -1[/tex]

[tex]f(1) = (1)^3 - 6*1 - 6 = -11[/tex]

[tex]f(2) = (2)^3 - 6*2 - 6 = -10[/tex]

[tex]f(3) = (3)^3 - 6*3 - 6 = 3[/tex]

[tex]f(6) = (6)^3 - 6*6 - 6 = 174[/tex]

For r to be rational;

The divisors of -6 must divide f(x) without remainder

i.e. Any of the above values  must equal 0

Since none equals 0, then r is irrational

Three less than 3 times a number, n, is
19 more than twice the number. What
is the number?
А
17
B
19
21
D 22

Answers

The equation is:

3n - 3 = 2n + 19

=> 3n - 2n = 19 + 3

=> n = 22

So, the answer is 22.

can someone help me with this please​

Answers

Answer:

for question B

2A 3C = 70

70-15 = 55/5 = 11.

so each child is 11

while adult 11 + 7.50 = 18.50

Alexandra finished 3/5th of her work. What percentage of work did she complete?

Answers

Answer:

60 percent

Step-by-step explanation:

Covert 3/5th into percentage so the answer will be 60.

#CarryOnLearning

Which of the following is a composite number?
A. 1
B. 63
C.O
D. 19

Answers

B. 63

some simple googling would've been able to help you with this. but 0 isn't prime or composite, not sure about 1, 19 is prime, 63 is a definite composite

Eva created a coffee blend for her restaurant by mixing Kenya and French Roast coffee beans. The Kenya beans cost $15 per pound and the French Roast costs $8 per pound. She bought a total of 10 pounds of coffee for a total of $97.50. Please help ASAP!!!!

Answers

Answer:

what is the question you are only telling us the fact

Step-by-step explanation:

Given the roll of paper towels below how much plastic would be needed to cover the role so it can be sold given the diameter of the role is 10 inches and the height is 13 inches

Answers

Answer:

Amount of plastic need to cover paper role = 565.2 inches

Step-by-step explanation:

Given:

Diameter of paper role = 10 inch

Height of paper role = 13 inch

Find:

Amount of plastic need to cover paper role

Computation:

Radius of paper role = Diameter of paper role / 2

Radius of paper role = 10 / 2

Radius of paper role = 5 inch

Amount of plastic need to cover paper role = Total surface area of cylinder

Amount of plastic need to cover paper role = 2πr(h+r)

Amount of plastic need to cover paper role = 2(3.14)(5)(13+5)

Amount of plastic need to cover paper role = (3.14)(10)(18)

Amount of plastic need to cover paper role = 565.2 inches

simplify (-9)×5×6(-3)​

Answers

[tex]\bf \large \hookrightarrow \:( - 9) \: \times \: 5 \: \times \: 6 \: \times \: ( - 3) \\ \\ \bf \Large \hookrightarrow \: \: \: - 45 \: \times \: ( - 18) \\ \\ \bf \Large \hookrightarrow \: \: 810[/tex]

Using trial and improvement, find the solution between 5 and 6 for the following equation:

x

2

=

27

Give your answer rounded to 1 DP.

Answers

Answer:

2

Step-by-step explanation:

that is √ 14 which is 27 ×>44.2177

2/5x - 1/5y = 98
2/7x - 1/14y = 55
If the ordered pair (x, y) satisfies the system of
equations shown above, what is the value of x?

Answers

Answer:

x = 140

Step-by-step explanation:

Given the equations

[tex]\frac{2}{5}[/tex] x - [tex]\frac{1}{5}[/tex] y = 98 ( multiply through by 5 to clear the fractions )

2x - y = 490 → (1)

[tex]\frac{2}{7}[/tex] x - [tex]\frac{1}{14}[/tex] y = 55 ( multiply through by 14 to clear the fractions )

4x - y = 770 → (2)

Subtract (1) from (2) term by term to eliminate y

2x + 0 = 280

2x = 280 ( divide both sides by 2 )

x = 140

i13(12)-2give answer​

Answers

154 I might be wrong tho

What is the inverse of the function f(x) = x +3?
O h(x) = 5x + 3
O h(x) =
1x-3
Oh(x) = x-3
Oh(x) = x + 3

Answers

I think it’s f^-1(x)=x-3

Explanation:

To find the inverse of a function, just "trade" x and y and solve for the "new" y. The graph of the inverse is the reflection of the original function over the line y=x.

The value of the inverse of the function f(x) = x +3 is,

⇒ h (x) = x - 3

What is mean by Function?

A relation between a set of inputs having one output each is called a function. and an expression, rule, or law that defines a relationship between one variable (the independent variable) and another variable (the dependent variable).

Given that;

Function is,

f (x) = x + 3

Now, We can find the inverse of the function f(x) = x +3 as,

f (x) = x + 3

y = x + 3

x = y - 3

h (x) = x - 3

Thus, The value of the inverse of the function f(x) = x +3 is,

⇒ h (x) = x - 3

Learn more about the function visit:

https://brainly.com/question/11624077

#SPJ7

Graph the solution of the inequality 3/7(35x-14)<_ 21x/2+3

Answers

Answer:

You'll have a closed circle at x = 2, and shading to the left

See the diagram below

=========================================================

Explanation:

The fractions here are 3/7 and 21/2. The denominators of which are 7 and 2 respectively. The LCD is 7*2 = 14.

If we multiply both sides by 14, then this will clear out the denominators and make the fractions go away.

14*(3/7) = (14*3)/7 = 42/7 = 814*(21/2) = (14*21)/2 = 294/2 = 147

So if we multiplied both sides by 14, then we have these steps

[tex]\frac{3}{7}(35x-14) \le \frac{21x}{2}+3\\\\14*\frac{3}{7}(35x-14) \le 14*\left(\frac{21x}{2}+3\right)\\\\14*\frac{3}{7}(35x-14) \le 14*\left(\frac{21x}{2}\right)+14*\left(3\right)\\\\6(35x-14) \le 147x+42\\\\[/tex]

--------------------------

Let's isolate x

[tex]6(35x-14) \le 147x+42\\\\6(35x)+6(-14) \le 147x+42\\\\210x-84 \le 147x+42\\\\210x-147x \le 42+84\\\\63x \le 126\\\\x \le 126/63\\\\x \le 2\\\\[/tex]

The graph of this will consist of a closed or filled in circle at x = 2. We shade to the left to represent numbers smaller than 2.

So either x = 2 or x < 2.

If we used an open hole at 2, then we wouldn't be including 2 (but we want to include this endpoint).

See the diagram below.

What is the midpoint of the segment shown below?

Answers

Step-by-step explanation:

simply take the average of the x and y coordinates

Una torre de 28.2 m de altura esta situada a la orilla de un rio, desde lo alto del edificio el ángulo de depresión a la orilla opuesta es de 25.2°. Calcular el ancho del río

Answers

Answer:

El ancho del río es 59.9 metros.

Step-by-step explanation:

El ancho del río lo podemos calcular con la siguiente relación trigonométrica asumiendo que la torre forma un triángulo rectángulo con el río:

[tex]tan(\theta) = \frac{CO}{CA}[/tex]

En donde:

CA: es el cateto adyacente = Altura de la torre = 28.2 m

CO: es el cateto opuesto = ancho del río =?

θ: es el ángulo adyacente a CA

Dado que el ángulo de depresión (25.2°) está ubicado fuera de la parte superior de la hipotenusa del triángulo que forma la torre con la orilla opuesta del río, debemos calcular el ángulo interno (θ) como sigue:

[tex]\theta = (90 - 25.2)^{\circ} = 64.8 ^{\circ}[/tex]

Ahora, el ancho del río es:

[tex]CO = tan(\alpha)*CA = tan(64.8)*28.2 = 59.9 m[/tex]

Por lo tanto, el ancho del río es 59.9 metros.

Espero que te sea de utilidad!                  

Can someone help me with this math homework please!

Answers

Answer:

x = 12.5

Step-by-step explanation:

Explanation in progress ;-)

Other Questions
What is potential energy? What are some of its examples. When was Dr. Raphael Ernest Grail Amattoe born ? Any help is much appreciated! What are emotive action or doing words?A. Active voiceB. ImperativeC. Powerful verbD. Passive voice Hy gip ti ln bi tp ny Lucy has been the sole shareholder of a calendar year S corporation since 1980. At the end of 2011, Lucy's stock basis is $23,500, and she receives a distribution of $25,000. Corporate level accounts are computed as follows.AAA 7,000PTI 11,000Accumulated E&P 600How much capital gain, if any, will Lucy have?a. $600b. $7,000c. $6,400d. $900 e. None of the above Prove:tan^3x/sin^2x-1/sinx.cosx+cot^3x/cos^2x=tan^3x+cot^3x The Columbian Exchange was a transfer of people, goods, and ideas between _____.the Eastern and Western hemispheresAsia and the AmericasAfrica and the Americasthe Northern and Southern hemispheres Please HelpSolve 3(x+2)-4x=8 (Please help)What is the most common isotope for element X A male Bengal tiger is mated with a female Bengal tiger. Orange (O) dominates with(o)coat color. A secondary concern is mating Bengal tigers with alleles for white coats is an occurrence of weak wrist bones. Normal wrist bones (B) dominates weak (b). Both parents are heterozygous for both traits. What is the genotype of the parents? OoBb OOBb OoBB oobb Find the volume of cuboid of side 4cm. Convert it in SI form Which agreement solved over whether salves should be counted for the purposes of representation What is the answer for this equation What is an amount between $2 and $10? Out of the four pillars of Aristotle's persuasion, which one has the potential to be the most powerful/most persuasive?Answer: LogosPathosEthosKairos How do forces between particles in gases compare to forces in the other states of matter? No trolling please. I will mark brainlist Which statements below represent the situation? Select three options. PLS HELP ASAP!!!solve for f(3) if f(x)=x^3+2x^2-x-1